22
$\begingroup$

For a real number $\alpha$, let the irrationality measure $\mu(\alpha) \in \mathbb{R}\cup \{\infty\}$ be defined as the supremum of all real numbers $\mu$ such that $$ \left| \alpha-\frac{p}{q}\right| \le \frac{1}{q^\mu} $$ has infinitely many solutions $\frac{p}{q} \in \mathbb{Q}$, where $q \ge 1$. It's known that if $\alpha$ is rational, then $\mu(\alpha)=1$ and if $\alpha$ is algebraic and irrational, then $\mu(\alpha)=2$ by Roth's Theorem. The set of all $\alpha$ such that $\mu(\alpha) >2$ has Lebesgue measure 0 by Khinchin's Theorem.

One can explicitly write down real numbers $\alpha$ such that $\mu(\alpha) = \infty$ (e.g. Liouville's constant) and transcendental real numbers $\alpha$ such that $\mu(\alpha)=2$ (e.g. $\alpha =e$, see also this thread). It's also possible to find upper bounds on $\mu(\alpha)$ for some real numbers such as $\pi$. But I don't know of a single example of a real number $\alpha$ whose irrationality measure is known, finite and greater than 2.

Question: Is there an example of a real number whose irrationality measure $\mu(\alpha)$ is known exactly and satisfies $2<\mu(\alpha)<\infty$?

$\endgroup$
2
  • 4
    $\begingroup$ I imagine if one "toned down" Liouville's construction and wrote $\alpha=\sum_{n=1}^\infty 10^{-\lfloor\mu^n\rfloor}$, then one could show that $\mu(\alpha)=\mu$. There is something to be shown, though, since the obvious rational approximations by terminating decimals only demonstrate that $\mu(\alpha)\ge\mu$. $\endgroup$ Jun 18, 2013 at 17:13
  • 1
    $\begingroup$ The Champernowne constant has irrationality measure $10$ (the base used in its construction). $\endgroup$ May 1, 2017 at 5:20

3 Answers 3

13
$\begingroup$

The answer is yes - see for example

Yann Bugeaud Diophantine approximation and Cantor sets Math. Ann. (2008) 341:677–684

$\endgroup$
3
  • 3
    $\begingroup$ I'm pmretty sure $\mu(\alpha)=1$ for rational $\alpha$ is correct. $\endgroup$ Jun 18, 2013 at 17:10
  • $\begingroup$ Sorry - corrected. $\endgroup$
    – user25199
    Jun 18, 2013 at 21:57
  • $\begingroup$ Thanks Carl, that's the kind of reference I was looking for. $\endgroup$
    – jsm
    Jun 19, 2013 at 17:29
15
$\begingroup$

Irrationality measure is a question about approximation by rationals. The continued fraction expansion gives the best approximations and controls their quality. Irrationality measure is a kind of asymptotic growth of the continued fraction expansion. Asking about the irrationality measure of a particular number is asking properties of its continued fraction expansion. But if you are willing to specify numbers by their continued fraction expansion, it is easy to write down a continued fraction expansion with the desired measure. Inductively define the continued fraction $[a_1,a_2,\ldots]$ by setting $a_{n+1}=\lceil q_n^{\mu-1}\rceil$, where the convergent is $\frac{p_n}{q_n}$. Then $q_{n+1}=a_{n+1}q_n+q_{n-1}$, so the error $(q_nq_{n+1})^{-1}$ is about $q_n^{-\mu}$.

It's a bit of a cop-out, but it's definitely worth mentioning.

$\endgroup$
2
  • $\begingroup$ Thanks, I'm happy to specify numbers by their continued fraction expansion. But don't you mean $\mu-2$ instead of $\mu-1$? (sorry if I'm being stupid here) $\endgroup$
    – jsm
    Jun 19, 2013 at 17:39
  • $\begingroup$ Yes, you're right, it should be $a_{n+1}=q_n^{\mu-2}$. Then $q_{n+1}\sim q_n^{\mu-1}$ and the reciprocal error is $q_{n+1}q_n\sim q_n^\mu$. $\endgroup$ Jun 19, 2013 at 18:39
7
$\begingroup$

The irrationality measure of the Champernowne constant $C_b$ in base $b>2$ is exactly $b$.

$\endgroup$

Your Answer

By clicking “Post Your Answer”, you agree to our terms of service and acknowledge you have read our privacy policy.

Not the answer you're looking for? Browse other questions tagged or ask your own question.